Prove $\sum{\frac{1}{(x+2y)^2}} \geq\frac{1}{xy+yz+zx}$

$a=x+2y,b=y+2z,c=z+2x \implies x=\dfrac{a-2b+4c}{9},y=\dfrac{b-2c+4a}{9},z=\dfrac{c-2a+4b}{9}$

the inequality become:

$\dfrac{1}{a^2}+\dfrac{1}{b^2}+\dfrac{1}{c^2} \ge \dfrac{27}{5(ab+bc+ac)-2(a^2+b^2+c^2)}$

now use UVW method:

$3u=a+b+c,3v^2=ab+bc+ac,w^3=abc \implies u\ge v\ge w$

$\iff \dfrac{(3v^2)^2-6uw^3}{w^6}\ge \dfrac{3}{3v^2-2u^2} \iff w^6+2u(3v^2-2u^2)w^3-3v^4(3v^2-u^2) \le 0$

let $w^3=x,f(x)=x^2+2u(3v^2-2u^2)x-3v^4(3v^2-2u^2)$

$2u(3v^2-2u^2) \ge 0 $

$f_{max}(x)=f(w^3|w=v)=f(v^3)$, when $w=v \implies u=v=w \implies f(v^3)=0 \implies f(x) \le 0 $

when $u=v=w \implies a=b=c \implies x=y=z$

QED.


Let $a+b+c=3u$, $ab+ac+bc=3v^2$, $abc=w^3$ and $u^2=tv^2$.

Hence, $t\geq1$ and we need to prove that $$\sum_{cyc}(4a^5b+4a^4c-12a^4b^2+12a^4c^2+5a^3b^3+8a^4bc-19a^3b^2c+5a^3c^2b-7a^2b^2c^2)\geq0$$ or $$\sum_{cyc}(4a^5b+4a^4c+5a^3b^3+8a^4bc-7a^3b^2c-7a^3c^2b-7a^2b^2c^2)\geq$$ $$\geq12\sum_{cyc}(a^4b^2-a^4c^2+a^3b^2c-a^3c^2b)$$ or $$\sum_{cyc}(4a^5b+4a^4c+5a^3b^3+8a^4bc-7a^3b^2c-7a^3c^2b-7a^2b^2c^2)\geq$$ $$\geq12(a-b)(a-c)(b-c)(a+b+c)(ab+ac+bc).$$ Since by Schur $3\sum\limits_{cyc}(a^3b^3-a^3b^2c-a^3c^2b+a^2b^2c^2)\geq0$, it's enough to prove that $$\sum_{cyc}(4a^5b+4a^4c+2a^3b^3+8a^4bc-4a^3b^2c-4a^3c^2b-10a^2b^2c^2)\geq$$ $$\geq12(a-b)(a-c)(b-c)(a+b+c)(ab+ac+bc)$$ or $$\sum_{cyc}(2a^5b+2a^4c+a^3b^3+4a^4bc-2a^3b^2c-2a^3c^2b-5a^2b^2c^2)\geq$$ $$\geq6(a-b)(a-c)(b-c)(a+b+c)(ab+ac+bc)$$ or $$18u^4v^2-24u^2v^4+5v^6+2u^3w^3-uv^2w^3\geq2(a-b)(a-c)(b-c)uv^2$$ and since $18u^4v^2-24u^2v^4+5v^6+2u^3w^3-uv^2w^3\geq0$, it's enough to prove that $$\left(18u^4v^2-24u^2v^4+5v^6+2u^3w^3-uv^2w^3\right)^2\geq4(a-b)^2(a-c)^2(b-c)^2u^2v^4$$ or $$\left(18u^4v^2-24u^2v^4+5v^6+2u^3w^3-uv^2w^3\right)^2\geq108(3u^2v^4-4v^6-4u^3w^3+6uv^2w^3-w^6)u^2v^4$$ or $$(4u^4-4u^2v^2+109v^4)u^2w^6+2uv^2(36u^6+150u^4v^2-290u^2v^4-5v^6)w^3+$$ $$+(324u^8-864u^6v^2+432u^4v^4+192u^2v^6+25v^8)v^4\geq0.$$ Since $324t^4-864t^3+432t^2+192t+25>0$, it's remains to prove that $$t(36t^3+150t^2-290t-5)^2-t(4t^2-4t+109)(324t^4-864t^3+432t^2+192t+25)\leq0$$ or $$t(t-1)^2(144t^3-72t^2-216t-25)\leq0,$$ where $t\geq1$ and $36t^3+150t^2-290t-5<0$.

Let $p(t)=36t^3+150t^2-290t-5$ and $q(t)=144t^3-72t^2-216t-25$.

By Descartes $p$ has an unique positive root $x_1$ and $q$ has an unique positive root $x_2$.

But $p(1.5)=19>0$ and $q(1.5)=-25<0$, which says that $q(x)<0$ for all $1\leq x\leq x_1$

and we are done!


Also we can make the following.

From my previous post we need to prove that: $$\sum_{cyc}(4a^5b+4a^4c-12a^4b^2+12a^4c^2+5a^3b^3+8a^4bc-19a^3b^2c+5a^3c^2b-7a^2b^2c^2)\geq0$$ or $$6\sum_{cyc}ab(a^2-b^2-2ab+2ac)^2+\sum_{sym}(2a^5b-a^3b^3-4a^4bc+10a^3b^2c-7a^2b^2c^2)\geq0,$$ which is obviously true.